2
0
Fork 0
mirror of https://github.com/MartinThoma/LaTeX-examples.git synced 2025-04-19 11:38:05 +02:00

cref: 'Satz' hinzugefügt; Lösung zu Aufgabe hinzugefügt; Bild hinzugefügt; Kleinigkeiten verbessert

This commit is contained in:
Martin Thoma 2014-01-13 20:33:39 +01:00
parent a33d36f8c7
commit 4d440a3624
8 changed files with 63 additions and 15 deletions

View file

@ -27,3 +27,4 @@ in dem Erstellen dieses Skripts steckt:
|12.01.2014 | 16:00 - 16:15 | `cleveref` benutzt
|12.01.2014 | 22:15 - 22:30 | Beweis erstellt, dass Überlagerungen surjektiv sind
|12.01.2014 | 23:30 - 00:00 | Gruppenaktion -> Gruppenoperation; Projektiver Raum zu Index hinzugefügt
|13.01.2014 | 19:00 - | TODOs erledigen

Binary file not shown.

View file

@ -762,7 +762,7 @@ $\qed$
Damit ist $K$ abgeschlossen. $\qed$
\end{beweis}
\begin{korollar}
\begin{korollar}\label{kor:5.6}%In Vorlesung: Bemerkung 5.6
Seien $X, Y$ topologische Räume, $f: X \rightarrow Y$ stetig.
Ist $K \subseteq X$ kompakt, so ist $f(K) \subseteq Y$ kompakt.
\end{korollar}
@ -779,7 +779,7 @@ $\qed$
Es gilt: $f(f^{-1}(V)) = V \cap f(X) \qed$
\end{beweis}
\begin{satz}[Heine-Borel]
\begin{satz}[Heine-Borel]\label{satz:heine-borel}%In Vorlesung: Proposition 5.7
Eine Teilmenge von $\mdr^n$ oder $\mdc^n$ ist genau dann kompakt,
wenn sie beschränkt und abgeschlossen ist.
\end{satz}

View file

@ -43,24 +43,25 @@
\item $\mdc^n$ ist eine $2n$-dimensionale Mannigfaltigkeit
mit einem Atlas aus einer Karte:
\[(z_1, \dots, z_n) \mapsto (\operatorname{Re} z_1, \operatorname{Im}z_1, \dots, \operatorname{Re}z_n, \operatorname{Im}z_n)\]
\item \xindex{Raum!projektiver}$\praum^n(\mdr) = (\mdr^{n+1} \setminus \Set{0})/_\sim = S^n /_\sim$ und $\praum^n(\mdc)$ sind Mannigfaltigkeiten \todo{besser aufschreiben}
der Dimension $n$ bzw. $2n$.
\item \xindex{Raum!projektiver}$\praum^n(\mdr) = (\mdr^{n+1} \setminus \Set{0})/_\sim = S^n /_\sim$ und $\praum^n(\mdc)$ sind Mannigfaltigkeiten
der Dimension $n$ bzw. $2n$, da gilt:
$\praum^n(\mdr) = \bigcup_{i=0}^n U_i,$
Sei $U_i := \Set{(x_0: \dots : x_n) \in \praum^n(\mdr) | x_i \neq 0}\;\forall i \in 0, \dots, n$.
Dann ist $\praum^n(\mdr) = \bigcup_{i=0}^n U_i$ und die Abbildung
\begin{align*}
U_i = \Set{(x_0: \dots : x_n) \in \praum^n(\mdr) | x_i \neq 0} &\rightarrow \mdr^n\\
U_i &\rightarrow \mdr^n\\
(x_0 : \dots : x_n) &\mapsto \left (\frac{x_0}{x_i}, \dots, \frac{x_i}{x_i}, \dots, \frac{x_n}{x_i} \right )\\
(y_1 : \dots : y_{i-1} : 1 : y_i : \dots : y_n) &\mapsfrom (y_1, \dots, y_n)
\end{align*}
ist bijektiv.
Die $U_i,\; i = 0, \dots, n$ bilden einen $n$-dimensionalen Atals.
\todo[inline]{Was wird im Folgenden gemacht?}
Die $U_i$ mit $i = 0, \dots, n$ bilden einen $n$-dimensionalen Atlas:
\begin{align*}
x &= (1:0:0) &y &= (0:1:1) \in U_2 \rightarrow \mdr^2\\
\in U_0 &\rightarrow \mdr^2 &y &\mapsto (0,1)\\
x &\mapsto (0,0) &&\text{Umgebung: } \fB_1 (0,1) \rightarrow \Set{(w:z:1) | w^2 + z^2 < 1} = V_2
x &= (1:0:0) \in U_0 \rightarrow \mdr^2 & x &\mapsto (0,0)\\
y &= (0:1:1) \in U_2 \rightarrow \mdr^2 & y &\mapsto (0,1)
\end{align*}
Umgebung $\fB_1(0,1) \rightarrow \Set{(1:u:v) | \|(u,v)\| < 1} = v_1$
$\text{Umgebung: } \fB_1 (0,1) \rightarrow \Set{(1:u:v) | \|(u,v)\| < 1} = V_1$\\
$\text{Umgebung: } \fB_1 (0,1) \rightarrow \Set{(w:z:1) | w^2 + z^2 < 1} = V_2$\\
$V_1 \cap V_2 = \emptyset$?
@ -363,8 +364,9 @@ $\partial X$ ist eine Mannigfaltigkeit der Dimension $n-1$.
\begin{definition}
$S \subseteq \mdr^3$ heißt \textbf{reguläre Fläche}\xindex{Fläche!reguläre} $:\gdw$
$\forall s \in S\;\exists $ Umgebung $V(s) \subseteq \mdr^3$ $\exists U \subseteq \mdr^2$ offen: $\exists$ differenzierbare Abbildung
$F: U \rightarrow V \cap S$: $\text{Rg}(J_F(u)) = 2\;\;\;\forall u \in U$.
$\forall s \in S\;\exists $ Umgebung $V(s) \subseteq \mdr^3$ $\exists U \subseteq \mdr^2$ offen:
$\exists \text{ differenzierbare Abbildung } F: U \rightarrow V \cap S$:
$\text{Rg}(J_F(u)) = 2\;\;\;\forall u \in U$.
$F$ heißt (lokale) reguläre Parametrisierung von $S$.

View file

@ -76,6 +76,13 @@ aufgestellt.
\end{enumerate}
\end{definition}
\begin{figure}[htp]
\centering
\input{figures/topo-halbgerade.tex}
\caption{Halbgeraden}
\label{fig:halbgeraden}
\end{figure}
\begin{korollar}
\begin{enumerate}[label=(\roman*)]
\item $PR^+ \cup PR^- = PR$

View file

@ -97,7 +97,32 @@
\end{solution}
\begin{solution}[\ref{ub3:aufg1}]
\todo[inline]{Kommt noch.}
\begin{enumerate}[label=(\alph*)]
\item \textbf{Beh.:} $\GL_n(\mdr)$ ist nicht kompakt.\\
\textbf{Bew.:} $\det: \GL_n(\mdr) \rightarrow \mdr \setminus \Set{0}$
ist stetig. Außerdem ist
$\det(\GL_n(\mdr)) = \mdr \setminus \Set{0}$ nicht
kompakt. $\stackrel{\ref{kor:5.6}}{\Rightarrow}$
$\GL_n(\mdr)$ ist nicht kompakt. $\qed$
\item \textbf{Beh.:} $\SL_1(\mdr)$ ist nicht kompakt, für $n > 1$ ist $\SL_n(\mdr)$ kompakt.\\
\textbf{Bew.:} Für $\SL_1(\mdr)$ gilt:
$\SL_1(\mdr) = \Set{A \in \mdr^{1 \times 1} | \det A = 1} = \begin{pmatrix}1\end{pmatrix} \cong \Set{1}$.
$\stackrel{\ref{kor:5.6}}{\Rightarrow} \SL_1(\mdr)$ ist
kompakt.\\
$\SL_n(\mdr) \subseteq \GL_n(\mdr)$ lässt sich mit einer
Teilmenge des $\mdr^{n^2}$ identifizieren. Nach \cref{satz:heine-borel}
sind diese genau dann kompakt, wenn sie beschränkt und
abgeschlossen sind. Definiere nun für für $n \in \mdn_{\geq 2}, m \in \mdn: A_m = \text{diag}_n(m, \frac{1}{m}, \dots, 1)$.
Dann gilt: $\det A_m = 1$, d.~h. $A_m \in \SL_n(\mdr)$,
und $A_m$ ist unbeschränkt, da $\|A_m\|_\infty =m \xrightarrow[m \rightarrow \infty]{} \infty$.$\qed$
\item \textbf{Beh.:} $\praum(\mdr)$ ist kompakt.\\
\textbf{Bew.:} $\praum(\mdr) \cong S^n/_{x \sim -x}$.
Per Definition der Quotiententopologie ist die Klassenabbildung stetig.
Da $S^n$ als abgeschlossene und beschränkte Teilmenge
des $\mdr^{n+1}$ kompakt ist $\stackrel{\ref{kor:5.6}}{\Rightarrow}$
$\praum(\mdr)$ ist kompakt. $\qed$
\end{enumerate}
\end{solution}
\begin{solution}[\ref{ub4:aufg1}]

View file

@ -0,0 +1,12 @@
\begin{tikzpicture}
\tikzstyle{point}=[circle,thick,draw=black,fill=black,inner sep=0pt,minimum width=4pt,minimum height=4pt]
\node (Pleft) at (0,0) {};
\node (P)[point,label=90:$P$] at (2,0) {};
\node (R)[point,label=90:$R$] at (4,0) {};
\node (Rright) at (6,0) {};
\draw[dashed,very thick] (Pleft) -- (P);
\draw[dotted,very thick] (P) -- (R) -- (Rright);
\draw [thick,decoration={brace,mirror,raise=0.2cm},decorate] (Pleft) -- (P) node [pos=0.5,anchor=north,yshift=-0.25cm] {$PR^-$};
\draw [thick,decoration={brace,mirror,raise=0.2cm},decorate] (P) -- (R) node [pos=0.5,anchor=north,yshift=-0.25cm] {$\overline{PR}$};
\draw [thick,decoration={brace,mirror,raise=0.8cm},decorate] (P) -- (Rright) node [pos=0.5,anchor=north,yshift=-0.85cm] {$PR^+$};
\end{tikzpicture}

View file

@ -89,3 +89,4 @@
% http://tex.stackexchange.com/a/101138/5645
\newcommand\rtilde[1]{\widetilde{\mathit{#1}}}
%%%%%%%%%%%%%%%%%%%%%%%%%%%%%%%%%%%%%%%%%%%%%%%%%%%%%%%%%%%%%%%%%%%%%
\crefname{satz}{Satz}{Sätze}